0 Daumen
716 Aufrufe

Aufgabe:$$\lim\limits_{n\to\infty} [7^{n}+3^{n}]^{\frac{1}{n}}$$

Folge berechnen



Problem/Ansatz: ich weiß, dass es gegen 7 läuft.

Kann mir jemand den Lösungsweg erläutern

Avatar von

1 Antwort

+1 Daumen
 
Beste Antwort

$$L=\lim\limits_{n\to\infty}[7^n+3^n]^{\frac{1}{n}}$$ Verwende die logarithmische Funktion, die monoton ist:$$\ln L=\ln \lim\limits_{n\to\infty}[7^n+3^n]^{\frac{1}{n}}$$$$\lim\limits_{n\to\infty}\ln \left([7^n+3^n]^{\frac{1}{n}}\right)$$$$\lim\limits_{n\to\infty}\frac{1}{n}\ln \left([7^n+3^n]\right)$$ $$\lim\limits_{n\to\infty}\frac{\ln \left(7^n+3^n\right)}{n}$$L'Hospital anwenden:$$\lim\limits_{n\to\infty}\dfrac{\ln\left(7\right){\cdot}7^n+\ln\left(3\right){\cdot}3^n}{7^n+3^n}$$ Kommst du nun allein weiter?

Zwangsfaktorisierung: $$\lim\limits_{n\to\infty}\dfrac{\ln\left(7\right){\cdot}7^n+\ln\left(3\right){\cdot}3^n}{7^n+3^n}$$ Klammere \(7^n\) aus:$$\lim\limits_{n\to\infty}\dfrac{7^n\left(\ln(7)+\frac{\ln(3)}{7^n}{\cdot}\left(\frac{3}{7}\right)^n\right)}{7^n\left(1+\left(\frac{3}{7}\right)^n\right)}$$$$\lim\limits_{n\to\infty}\dfrac{\ln(7)+\frac{\ln(3)}{7^n}{\cdot}\left(\frac{3}{7}\right)^n}{1+\left(\frac{3}{7}\right)^n}=\frac{\ln(7)+0+0}{1+0}=\ln(7)$$ "Rücksubstitution":$$\ln(L)=\ln(7)  \quad \longrightarrow L=7$$

Avatar von 28 k

Nein ich hab keine Ahnung wie ich das weiter rechnen soll.

Ok, dann jetzt vollständig in der Antwort.

Vielen Danke

Gern geschehen! Hast du alles verstanden? Du kann ruhig fragen, ich habe im Moment ausnahmsweise Zeit.

Ein anderes Problem?

Stell deine Frage

Willkommen bei der Mathelounge! Stell deine Frage einfach und kostenlos

x
Made by a lovely community